Inscription / Connexion Nouveau Sujet
Niveau exercices
Partager :

Inégalité de Hölder

Posté par
Skops
30-05-08 à 19:34

Bonjour,

Démontrer que :

4$\rm\forall (a_1,..,a_n)\in\mathbb{R}^{*n}_+\\\forall (b_1,..,b_n)\in\mathbb{R}^{*n}_+\\p,q>0 \frac{1}{p}+\frac{1}{q}=1

5$\fbox{\bigsum^{n}_{k=1}a_kb_k\le(\bigsum^n_{k=1}a_k^p)^{\frac{1}{p}}(\bigsum^n_{k=1}b_k^q)^{\frac{1}{q}}}

Bonne chance

Skops

Posté par
infophile
re : Inégalité de Hölder 30-05-08 à 19:55

Bonsoir

 Cliquez pour afficher

Posté par
Skops
re : Inégalité de Hölder 30-05-08 à 20:00

Salut info

>>

 Cliquez pour afficher


Skops

Posté par
gui_tou
re : Inégalité de Hölder 31-05-08 à 15:56

A quand Cauchy-Schwartz ?

Posté par
lyonnais
re : Inégalité de Hölder 31-05-08 à 17:10

Cauchy Schwarz :D

Posté par
gui_tou
re : Inégalité de Hölder 31-05-08 à 17:13
Posté par
infophile
re : Inégalité de Hölder 31-05-08 à 17:13
Posté par
gui_tou
re : Inégalité de Hölder 31-05-08 à 17:13
Posté par
infophile
re : Inégalité de Hölder 31-05-08 à 17:14

Une fois c'est suffisant

Posté par
gui_tou
re : Inégalité de Hölder 31-05-08 à 17:17

Roo y a eu un bug

Posté par
lyonnais
re : Inégalité de Hölder 31-05-08 à 18:17

Bien vu gui_tou

Posté par
fusionfroide
re : Inégalité de Hölder 01-06-08 à 00:42

Allez je me lance ^^ Ca me fait des révisions

 Cliquez pour afficher


Question : démontrer l'inégalité de Minkowski à partir de l'inégalité de Hölder ...



Vous devez être membre accéder à ce service...

Pas encore inscrit ?

1 compte par personne, multi-compte interdit !

Ou identifiez-vous :


Rester sur la page

Désolé, votre version d'Internet Explorer est plus que périmée ! Merci de le mettre à jour ou de télécharger Firefox ou Google Chrome pour utiliser le site. Votre ordinateur vous remerciera !